Search found 300 matches


I believe the correct answer should be

by khanshainur

Mon May 16, 2016 12:03 am
Forum: Critical Reasoning
Topic: resolve the paradox
Replies: 17
Views: 5019

I would go with option D as the correct option

by khanshainur

Mon May 16, 2016 12:02 am
Forum: Critical Reasoning
Topic: People in isolated rain-forest
Replies: 8
Views: 3362

I am pretty sure that the right Answer is C

by khanshainur

Mon May 16, 2016 12:00 am
Forum: Critical Reasoning
Topic: Lsat 28 1.20
Replies: 6
Views: 1840

I believe the answer should be D

by khanshainur

Sun May 15, 2016 11:58 pm
Forum: Critical Reasoning
Topic: Help for right answer
Replies: 10
Views: 2238

I would go with option A as the correct option

by khanshainur

Sun May 15, 2016 11:58 pm
Forum: Critical Reasoning
Topic: tricky 1
Replies: 6
Views: 2022

I will Go with option C in this case

by khanshainur

Sun May 15, 2016 11:57 pm
Forum: Critical Reasoning
Topic: Elderly citizens
Replies: 6
Views: 6571

It seems to me that the right answer is C

by khanshainur

Sun May 15, 2016 11:56 pm
Forum: Critical Reasoning
Topic: Ergonomic Society
Replies: 8
Views: 4113

I’d be shocked if it wasn’t D

by khanshainur

Sun May 15, 2016 11:55 pm
Forum: Critical Reasoning
Topic: A recent university study
Replies: 11
Views: 2971
by khanshainur

Sun May 15, 2016 11:54 pm
Forum: Critical Reasoning
Topic: blackbirds stop - pls comment and answer
Replies: 12
Views: 2491

Don’t have much explanation for it, D seems good

by khanshainur

Sun May 15, 2016 11:53 pm
Forum: Critical Reasoning
Topic: Evaluate the arg
Replies: 2
Views: 1346

I would go with option B as the correct option

by khanshainur

Sun May 15, 2016 11:53 pm
Forum: Critical Reasoning
Topic: Martin’s Cafe
Replies: 10
Views: 4834

I will Go with option C in this case

by khanshainur

Sun May 15, 2016 11:52 pm
Forum: Critical Reasoning
Topic: Azor Language Institute
Replies: 7
Views: 2174

I believe the answer should be D

by khanshainur

Sun May 15, 2016 11:51 pm
Forum: Critical Reasoning
Topic: CR - Weaken the argument
Replies: 5
Views: 2141

I am pretty sure that the right Answer is E

by khanshainur

Sun May 15, 2016 11:51 pm
Forum: Critical Reasoning
Topic: Oligarchy is a Government
Replies: 5
Views: 2010